You are on page 1of 44

Steinman

Civ Pro I , Fall 2016


Civ Pro: Cases and Materials, 11th, Friendenthal et al.

Personal Jurisdiction
1. Early/Initial Development of Personal Jurisdiction
a. Types of Personal Jurisdiction Traditional Bas
b. Pennoyer v. Neff
i. A party may enter a judgment against a nonresident only if the party:
1. Is personally served with process while in the state (in personam)
2. Has tangible piece of property within the state, and that property is brought under
the control of the court (aka attached) before litigation begins. [in rem, quasi in
rem concerns a person, but jurisdiction limited to their property]
ii. This method is too rigid and no longer used by itself.

c. Personal Jurisdiction Expands [Hess v. Pawloski]


i. In rem and in personam no longer needed.
ii. States can constitutionally declare personal jurisdiction with State Long Arm Statutes.
1. E.g. One that automatically forces implied consent once driving on state highways.
2. Explicit v. Implicit consent negligible.

2. Specific Jurisdiction - Minimum Contacts Analysis


a. In order to have personal jurisdiction:
i. Must comply with states long-arm Statute and
ii. Not violate Due Process Clause of 14th Amendment
b. Does the long-arm capture defendant? (even in Fed. Court: Rule 4(k)(1)(A))
i. If long-arm extends to Constitution: The CA long-arm statute extends to the limits of the
US Constitution. Therefore the long-arm and constitutional inquiry become one [instead of
the two above].
c. International Shoe
i. Minimum contacts such that the suit does not offend traditional notions of fair play and
substantial justice.
ii. Reasonably expected to be haled into the forum.
d. McGee
i. One contact with substantial connection can still satisfy traditional notions of fair play
and substantial justice
1. CA had a statute specifically exercising jurisdiction over insurance company
2. derived revenue directly from McGee in CA
3. s action created the contact (purchasing other insurance company)
e. World-Wide Volkswagen A partys unilateral action that creates a contact with the forum does not
establish s minimum contacts.
i. Foreseeability (stream of commerce) insufficient to establish minimum contacts.
ii. Fair play and substantial justice 5-factor test:

1
1. Burden on defendant
2. s interest in obtaining convenient and effective relief when that nterest is not
adequately protected by the s power to choose the forum
3. Forum states interst in adjudicating the dispute
4. Interstate judicial systems interest in obtaining efficient resolution
5. Shared interest of the several states in furthering fundamental substantive social
policies.
f. Keeton: The plaintiff does not need minimum contacts, only the defendant.
g. Kulko: Father allowed daughter to move to CA w/ mother
i. Court invoked public policy reasons for declining jurisdiction would impose
unreasonable burden on family relations.
h. Calder: Calder Effects Test: jurisdiction over [defendants] is proper in CA based on the effects
of their Florida conduct in California.
i. CA was focal point of story and harm suffered.
ii. The defendants had acted intentionally produce an article for dissemination in CA.
i. Burger King: Choice of Law Clause very suggestive of purposeful availment but NOT dispositive
i. Other factors: long-term relationship, payments and notices to the forum, corporation
exercising control over defendant from the forum, etc.
ii. Contact is not random, fortuitous as it is in Hanson.
j. Asahi: Forum state had no interest in the suit, foreign relations also played a role, no fair play and
substantial justice.
i. OConnor stream of commerce PLUS act purposefully directed at forum state
ii. Brennan stream of commerce: refers to regular and anticipated flow of products
(foreseeability is key)
k. McIntyre: Even stricter than OConnors stream of commerce plus
i. Justice Kennedy rejects the idea of a natl forum, must conduct a forum-by-forum analysis
to determine minimum contacts.
1. The Principle inquiry in cases of this sort is whether the defendants activities
manifest an intention to submit to the power of a sovereign [] the defendant must
purposefully avail itself of the privilege of conducting activities within the forum
state, thus invoking the benefits and protections of its laws.
l. Internet and Other Technological Contacts
i. Zippo: Sliding scale of interactivity determining whether a defendant is subject to
jurisdiction in the forum
m. Quasi In Rem Jurisdiction: you must be able to conceptualize the property in the state to attach it.
Harris.
i. Property within the state PLUS minimum contacts analysis. Shaffer
ii. Only useful if long-arm does not extend to bounds of Constitution
iii. Cause of action does NOT need to arise out of the property [?]
iv. Limited Appearance: Allows a defendant in an action quasi in rem to defend on the
merits without submitting to the jurisdiction of the court in person.
n. Rule 4(k) contains special service rules.
o. CHALLEGING PERSONAL JURISDICTION:
i. Special appearance

2
ii. Collateral attack: do not appear, then collaterally attack default judgment
1. If jurisdiction has been litigated, cannot re-litigate [res judicata].

2. Contact must be with the state itself, not residents of the state.

iii. Purposeful Availment


1. Examples:
a. World-Wide Volkswagen, 1980 NY corporation sold car in NY and the
car was wrecked in Oklahoma. Oklahoma court was not granted
jurisdiction because defendant corporation had not availed itself of the
privilege of conducting activities within the forum State. Specific
jurisdiction cannot be established when connection with forum state is the
result of a consumers unilateral activity. did not sell cars, did not
advertise, did not cultivate OK customers, and did not deliberately focus
on OK as market. So, didnt seek direct benefit of laws - cant be required
to submit to jurisdiction.

b. J. McIntyre, 2011 New Jersey man injured his hand using a machine
manufactured by J. McIntyre in England and shipped from companys US
distributor. J. McIntyre targeted entire US, but not forum state. No binding
decision, court was split; ruled that J. McIntyre did not purposefully avail
itself of the privilege of conducting activities within the forum State
because company did not target the forum. No jurisdiction.

c. Walden, 2014 Defendant performed improper search and seizure of


plaintiffs property in Georgia. Plaintiff returned home to Nevada and sued
defendant in Nevada court. Defendant had no connection to Nevada except
that plaintiff was from Nevada. Court ruled that the plaintiff couldnt be
the only link between the defendant and the forum.

2. Stream of Commerce: When is putting product into stream of commerce sufficient


to establish jurisdiction?

a. Placement: Merely placing item into stream of commerce is not enough


[although Gray, 1961 ruled mere introduction was sufficient, it was
overruled later].

b. Knowledge v. Intent: It is unresolved whether putting product in the


stream of commerce with the knowledge that it will end up in forum state
is sufficient, or whether one must target the forum state. Conduct a
fairness analysis here.
i. Examples:

3
1. Asahi Metal, 1987 Foreign corporation manufactured
tire valves and shipped them to different foreign
corporation which placed valves in tires and shipped 20
percent of those tires to forum state. No binding decision,
court was split; debate was over whether corporations
knowledge that thousands of its product will be used in
forum state qualifies as purposeful availment.

2. Ainsworth, 2013 Rule of Law: Using the stream-of-


commerce approach to justify personal, specific
jurisdiction does not contradict the Supreme Court's
holding in McIntyre and thus is sufficient to satisfy due
process. McIntyre did not change the law or alter
jurisdictional framework in determining personal
jurisdiction.
a. Court ruled that forum state is granted jurisdiction
by minimum contacts standard when the court
finds that the defendant delivered the product
into the stream of commerce with the expectation
that it would be purchased by or used by
consumers in the forum state, meaning that the
defendants contacts must be more than random,
fortuitous, or attenuated, or of the unilateral
activity of another party or third person.

3. Contracts: A contract alone is insufficient to establish jurisdiction under due


process clause. A contract with substantial connection is sufficient.
a. McGee v. Intl Life Insurance Co. A state's long-arm statute can make a
foreign company amenable to personal jurisdiction using the insurance
contract as a minimum contact.
i. Consider (a) prior negotiations and (b) contemplated future
consequences.
ii. Why was contract substantial?
1. Delivered in CA, premiums were mailed from CA, and
insured was resident of CA when he died; CA has
undeniable interest in providing way of redress for
residents, who would be at severe disadvantage if they had
to make the claim in distant state [cost]; Crucial witness
also in McGee's state; Inconvenience to insurer does not
amount to denial of due process.
2. Granted CA jurisdiction.
b. Burger King, 1985 - Defendant entered into a 20-year contract with Burger
King that envisioned a broad and regular contact with Burger King in
Florida. Furthermore, he agreed to make send all relevant notices and

4
payments to Burger Kings Miami headquarters. Moreover, provisions in
contract provided that Florida law would govern all disputes. Contract was
ruled sufficient for jurisdiction.

4. Foreseeability: relevant to jurisdiction when defendants conduct and connection


with the state are such that he should reasonable anticipate being haled to court
there.
a. Example:
i. Keeton v. Hustler Jurisdiction is supported even though s acts
had greater impact in other states and had few contacts with
forum state. SCOTUS held: Where a corporation that publishes a
magazine continuously and deliberately exploits a state's market, it
must reasonably anticipate being haled into court in the state in a
libel action based on the contents of its magazine

p. General v. Specific Jurisdiction


i. General Jurisdiction: Jurisdiction over a person/entity. Therefore, jurisdiction over all
matters. To establish general jurisdiction, the corporations dealings with the state must be
so continuous and systematic as to render it essentially at home in the forum state.
(Daimler, 2014)
ii. Specific Jurisdiction: Jurisdiction over a person/entity concerning actions performed within
the state. To establish specific jurisdiction, a corporations dealings with the forum state
must be continuous and systematic.

Specific General
Defendant's Contacts Lawsuit must arise from or relate to D's contacts with the forum
the defendant's contacts in the forum state must be so continuous
state. and systematic as to render it
essentially at home there.
Reasonableness/Fairness Separate inquiry is required. No separate inquiry is
Jurisdiction can be unconstitutional required.
even if has sufficient minimum If s contacts meet the
contact in the forum state. (e.g., high standard of general
Asahi) jurisdiction, jurisdiction
is constitutional.

Federal Rules of Civil Procedure & Personal Jurisdiction


1. Personal Jurisdiction in Federal Court: Rule 4(k)
a. 4(k)(1)(A): Federal Court has jurisdiction when state court where it sits has jurisdiction. So, in
most cases, the test for jurisdiction will be the same as if it were taking place in a state court.

5
i. Jurisdiction must be authorized by state law (long-arm statute)
ii. Jurisdiction must comply with the 14th Amendment (Due Process)
b. 4(k)(1)(C): Federal Court can exercise jurisdiction if a federal statute authorizes it. (e.g. a
nationwide service of process statute)
i. Example:
Panama v. BCCI Holdings Action filed under federal statute by a foreign
company, but basis of contact was with the U.S. as a whole, not an individual state.
Court held that Due Process cannot be satisfied with just contact as a whole
fairness and reasonableness should also be considered as in state court
jurisdictional decisions. Why? 5th Amendment (Fed. Due Process) & 14th
Amendment (State Due Process long-arm statutes) very similar language, but no
explicit precedent for 5th Amdmt, so courts should use 14th Amdmt precedent for
federal due process decisions. Factors determining fairness under 14th Amdmt
should also be used here.
Process:
i. Balance burdens imposed on the against federal interest
in litigation (Asahi)
ii. Infringement on liberties must be justified by federal
interest
iii. But not mechanically applied! *Only if extremely unfair*
c. 4(k)(2): Provides for federal jurisdiction if the is not subject to any state jurisdiction for matters
arising under federal law.
i. Example:
ISI Intl Inc. Borden Gervais Foreign company did not have substantial contact
with any one state over another, but did have contact with U.S. as a whole. Court
rule that under FRCP 4(k)(2) the Ill. Fed Court had jurisdiction over them [b/c the
court represented USA as a whole]. Jurisdiction counts if states he cant be sued
in the forum state but does not offer another state in which jurisdiction is present.
Constitution and Lanham Act must still authorize though [4k(2)(B)].

2. Service of Process in Federal Courts: Delivery of Summons and Complaint


a. Notice: Notice of a lawsuit must satisfy Due Process requirements.
i. Must reasonably convey information and give reasonable time for interested parties to
make an appearance.
1. If these conditions are reasonably satisfied given the specific circumstances of
case, the notice is constitutional.
2. Publication is not reliable means of notice, because chance of actual notice is low.
3. But when people are missing or address is unknown, indirect/futile notification
(publication) is constitutional.
ii. A party can appoint an agent not personally known to the other party to receive service of
process w/o violation of Due Process
iii. Rule 4(c): Delivery of summons and complaint must comply with following rules:
1. Who may serve: 18+, non-party, at plaintiffs request, marshal or someone
appointed.

6
2. Rule 4(d) - Waiving Service: 30 days from when request was sent, 60 days if
outside the U.S.
a. Advantages: It eliminates any initial, upfront cost of personal service on
you (even though the defendant would pay later, it still wouldn't require
you to spend any money at the outset, with no idea of when you will get it
back because you don't know how long the case will take to settle). It also
eliminates the hassle of finding someone to reliably personally serve it.
b. Disadvantages: The defendant, if they return the waiver timely, gets longer
to answer the complaint - 60 days if domestic and 90 days if in foreign
country - than if served with personal summons. (30 days). Gives them
more time to plan/formulate defense. It also, because it is purely by mail,
isn't as reliable as personal service and the waiver could get lost in either
direction, which would lead to complications and miscommunication.
Might stretch out process

3. How service is made:


a. Domestic individuals Rule 4(e)
i. Personally serving copy of summons + complaint to individual
ii. Leaving copy at house with someone 18+ who lives there
iii. Delivering copy of each to agent authorized by apt or by law
iv. Examples:
National Development Co., 1990 service was effected via
delivery to one of defendants twelve homes during a time he was
living in the home. It was accepted as reasonably calculated to
provide actual notice of the action.

National Equipment Rental, 1964 A contract between plaintiffs


corporation and defendant individual named a third party the agent
for service of process. The agent was unaware of her agency until
the suit and she was related to the plaintiff. Service was ruled to be
acceptable from the legally appointed agent, but affirmed that her
agency would have been questionable had she neglected to deliver
service to the defendant.

Insurance Co. of North America, 1980 the agent was not


expressly/formally authorized by defendant, but was ruled valid
because recipient was an employee, agreed to be the agent, and
had received process for the company before. This rendered it
fair, reasonable, and just to imply the authority on his part to
receive services, even though he lost the service to the ultimate
attachment of defendant corporations account by the court for
failure to reply to the summons.

b. Foreign Individuals Rule 4(f):

7
i. According to the international standards set forth in the Hague
Convention. *e.g. U.S. cannot mandate foreigner to pay for service
if they fail to waive service*
ii. By a method reasonably calculated to give notice:
1. According to that countrys laws
2. According to a foreign authoritys directions
3. By personal delivery or mail if allowed by the countrys
law
c. Corporation, Company Rule 4(h) authorizes service under 4(e)(1) with
exception of personal delivery/mail.
iv. Time Limit for Service Rule 4(m): must be served within 90 days after complaint is
filed, or the court on its own or by motion of , must dismiss or order service to be made in
specified time. Does not apply to service in a foreign country.
v. Immunity from Process: The federal courts recognize the immunity from service of process
of parties, witnesses, and attorneys who enter a state to appear in another action. In
addition, if a party was induced by the plaintiffs fraud or deceit to enter a state so that he
could be served, the service is invalid and the court does not acquire personal jurisdiction.

Subject Matter Federal Jurisdiction (Basics)


1. Subject-Matter Jurisdiction In General: power of a court to hear a case given the nature of the
dispute, distinct from its power to enter a judgment against a particular defendant.
a. State courts are courts of general jurisdiction meaning that they have complete, inherent power
and authority to hear cases on all subject matter unless they are ousted of such cases by state or
federal law. Jurisdiction determined by:
i. State constitution
ii. State statutes, and
iii. Judicial decisions
b. Federal courts are courts of limited subject matter jurisdiction meaning that they only have power
to hear a case if they are given that power by the following that determine jurisdiction:
i. Article III
ii. Federal Statutes, and
iii. Judicial decisions
c. State & Fed courts have overlapping jurisdiction (sometimes) that can allow to forum shop
among courts of diff. systems.
i. 28 U.S.C. 1441(a) allows for limited right of to transfer a case from state to federal
court. If civil action brought in a State court of which the district courts have original
jurisdiction, D can remove jurisdiction from the State by invoking this statute.
(a) If removes State Federal, can try and Federal State.
(b) cannot remove: Federal State
2. Federal Subject Matter Jurisdiction [Includes Federal Question Jxn + Diversity Jxn]
a. FEDERAL QUESTION JURISDICTION: 1331 provides that federal courts have original
jurisdiction of:
i. All civil actions arising under Constitution, laws, and treaties of U.S.

8
ii. 1331 is the JURISDICTIONAL STATUTE. NOT THE SUBSTANTIVE STATUTE. A suit
cannot arise out of federal law with just 1331 as the basis. Must have another
justification. (substantive federal question, not another procedural statute)
b. DIVERSITY!! 1332(a): Basics
i. AIC: Civil actions where matter in controversy exceeds sum/value of 75,000.
(a) burden: party invoking diversity jurisdiction has burden of showing that the
amount-in-controversy requirement has been met.
(b) Test: the sum claimed by the plaintiff controls if the claim is apparently made in
good faith therefore it must appear to a legal certainty that the claim is really for
less than the jurisdictional amount to justify dismissal.
a. Single Plaintiffs: AIC can be met in single - Single
suit by aggregating the value of all the claims against the , even if they
are completely separate and have no relation in law or fact w/ each other.
b. Multiple Plaintiffs: AIC can be met in Multiple s - Single only by
aggregating the value of claims that are common and indivisible.
i. Test:
1. Whether multiple parties seek "to enforce a single title or
right" e.g. if one P were to fail to collect the share, the rest
of the P's would split it.
2. If claims are joint and severable, can't be aggregated for
jurisdictional purposes.
ii. Civil Controversies between parties must have diversity, which is established when suit
is
(a) Between citizens of different states,
(b) Citizens of a State and citizens/subjects of a foreign state [exception: if someone is
a citizen of foreign state that has lawful permanent resident here and is domiciled
in same state as other party, diversity is destroyed.]
(c) This is known as complete diversity: no diversity jurisdiction if ANY is a
citizen of the same state as ANY , no matter how many parties are involved in
the litigation. (CJ Marshall, 1806)
(d) Held to be interpretation of statute, not a constitutional limitation and so a
powerful precedent.
*Note: Diversity jurisdiction is meant to discourage bias against out-of-state
citizens*
iii. Domicile Test: A party is domiciled in the state where his/her true, fixed, and permanent
home is located. Domiciliary changes only when
(a) Party takes up residence in new location AND
(b) Intends to stay at that location.
(c) Also domicile is determined at the point in time the complaint is filed, not when an
injury/accident/occurrence happened.
*Marriage does not automatically create domicile*
iv. Exception: Class Action [Title 28 1332(d)(2).]
v. Exception: Divorce, alimony, child custody cases; probate wills.
c. Title 1332 Diversity for Corporations

9
i. A corporation is a citizen of every State and foreign state that its incorporated in and of
the state/foreign state where it has principal place of business.

3. Federal Question Jurisdiction


a. According to 28 U.S.C. 1331, federal district courts have jurisdiction of all civil actions arising
under the Constitution, laws, or treaties of the United States.
i. Congress has the power to grant federal jurisdiction over any case that might call for the
application of federal law, i.e. if federal law could make or break the case. (Osborne,
1824)
(a) *Note: This includes for foreign plaintiffs (Verlinden, 1983)*
ii. Mottley Rule says that courts assess the federal court's subject matter jurisdiction based
solely on the P's complaint. (Not possible defenses, not counterclaims, etc.)
iii. GRABLE TEST: For state law claims with a federal ingredient embedded in them: the
question is: does a state-law claim necessarily raise a stated federal issue, actually
disputed and substantial, which a federal forum may entertain without disturbing any
congressionally approved balance of federal and state judicial responsibilities.? (Grable
& Sons, 2005)
(a) The effect on the federal judiciary is relevant here
(b) This does not include cases in which the only relevant federal law is the possibility
of a federal defense. (Mottley, 1908)
STATE LAW CAUSE OF ACTION
+ FEDERAL DEFENSE
1331 (fed question jxn)
iv. Examples:
(a) **Mottley Rule** Mottley, 1908 Diversity jurisdiction was not granted, because
the only federal law relevant to the couples suit was the possibility that the
railroad company would use the congressional Act as a defense.

(b) Kansas City Title, 1921 Federal statute, and parties are disagreeing about its
validity. Federal jurisdiction needed to clear it up in order for to have the right to
relief. Case held to be 1331 jurisdiction because the right to relief depended on
the construction or application of the constitution or laws of the U.S if the court
didnt grant jurisdiction to fed court, the would not have any way to get relief
[fed court was only way]. Basically, : I need to prove this federal ingredient in
order to prove my state law claim.

(c) Merrell Dow, 1986 Plaintiff sued Corporation over a violation of federal drug
labeling requirements. Court ruled that since plaintiff was not explicitly granted a
private right of action by congress to enforce federal labeling requirements and
since there was insufficient federal interest at stake, jurisdiction was improper.
(d) Moore v. Chesapeake & Ohio - Whether federal jurisdiction is available in an
action that arose under a state employer liability act that said that cannot use

10
violation of state/fed safety laws. Court held that just b/c the statute included
federal laws in its contents does not mean that the suit arose under laws of U.S. in
absence of diversity of citizenship [1332].

(e) Grable & Sons, 2005 Plaintiff had delinquent taxes and sued the IRS for failure
to serve process in exact accordance with a federal statute, and therefore contested
the seizure and sale of his property. Federal jurisdiction was ruled to be proper.
Rule: A federal court can have jurisdiction of a claim arising under a state
cause of action if the action necessarily raises a substantial federal component
that is actually disputed and in controversy, and if federal jurisdiction over that
claim would not disrupt congressionally-created balance between state and
federal courts.

Venue & Forum Non Conveniens


1. Venue: Think of venue as horizontal selection rather than vertical. Which venue can a suit be brought
within a particular legal system [aka state courts, federal courts]
a. Diversity Cases: 1391(a)
i. Except otherwise provided by law, venue is only proper in:
(a) A judicial district in which any resides, if all s are residents of State that
district is located;
a. individuals: residence = domicile
b. entities/corporations: see 1391(c)(2) below.
(b) A judicial district in which a substantial [important or significant] part of events or
omissions giving rise to claim occurred;
a. if property @ issue: venue is where substantial part of property is located
b. Venue is proper under for any district where a substantial part of the
events giving rise to the claim took place, even if substantial events also
took place in other districts. We don't have to choose which events are
most substantial.
(c) [FALLBACK PROVISION]. If a proper venue does not exist based on the first
two tests, then venue is proper wherever any defendant can be subject to personal
jurisdiction in the action.
a. **Only use when neither of the first two tests gives you any venue** aka it
will only be available when (a) the defendants do not all reside in the same
state for venue purposes and (b) all substantial parts of the events or
omissions giving rise to the claim occurred outside of the United States.
b. If either gives you a venue, then you have a venue, regardless if its the
venue you want, and you cant use the fallback provision.
c. When first two tests lay venue in districts besides the one where the case
has been filed, venue is improper in that district and the case should be
dismissed or transferred.
ii. If s reside in different districts but all within the same state, venue is proper in any
district in which one resides in that state.

11
b. Corporations: 1391(c)(2)
iii. A corporation resides,
(d) If a defendant, in any judicial district in which the is subject to the courts
personal jurisdiction for civil action in question
(e) If a plaintiff, only in the judicial district in which it maintains its principal place of
business.
c. 1391(d) Residency of corporations in states with multiple federal districts.
iv. Each district is treated as if it were a different state, so if corporation has personal
jurisdiction if the district were a separate state.
v. If there is not a district that corporation would have personal jurisdiction in, then use
district within which it has the most significant contacts.

2. Transfer of Venue
a. Always horizontal, so state-to-state and federal-to-federal, never state-to-federal or vice versa.
b. Transfers can only be made to districts where the case could have been originally brought, unless
all parties have consented to suit in the alternate forum.
c. 28 U.S.C. 1404: Courts take a broader reading of 1404 (transfer of venue) and considers such
factors as:
1) Access to witnesses
2) Access to forum
3) Docket congestion
4) Speed to trial
5) Relationship of the community to the dispute
6) The court's familiarity with the governing law
7) Plaintiff's forum choice
8) Whether the original forum is where plaintiff resides
9) The existence of a pre-suit venue agreement
b. Factors that are considered when deciding whether to transfer:
1) The availability and convenience of the witnesses and parties
2) The location of counsel
3) The location of books and records
4) The cost of obtaining attendance of witnesses and other trial expenses
5) The place of the alleged wrong
6) The possibility of delay and prejudice if transfer is granted
7) Plaintiff's choice of forum

12
Statute Section Transfer Purpose Mechanics Substantive Law Changes [Choice
of Law]

1404(A): Way to ensure Transfer is not compulsory, and court Transferee (new) court has to
convenience of cannot dismiss because current venue is follow the state laws of the original
change of
parties, non-biased proper. (transferor) court the suit was
venue courts, local issues Based on convenience of parties + brought in. This is to ensure the
(p. 219) stay local, etc. witnesses and in the interest of justice, change of venue stays just a
courts MAY transfer to adequate "change of courtrooms" and
alternative venue for the sake of change convenience -- not a change of
of venue, where suit: substantive laws. (Van Dusen Rule,
1. Might have been brought, or Note 4, p. 257). Essentially it keeps
2. To any district/division to which change of venue from becoming a
all parties consent change of jurisdiction.

1406(a): Makes up for a Dismissal OR transfer compulsory. Note 8, p. 259: Court will treat
[lawyer's] mistake Transferor court has power to transfer action as if it were started in
cure or
in filing a case in the even if it is not within personal transferee (destination) court.
waiver of wrong venue. jurisdiction (Note 9, p. 259). Court
defects cannot do nothing.
(p. 220) Can only dismiss if there is no other
proper venue to transfer. If another
venue option, court MUST transfer case
to venue where suit might have
originally been brought.

1631: Cure when an Transfer compulsory if it is in the interest Action proceeds in new venue as if
action is filed in a of justice. it had been originally filed there, so
transfer to court that doesn't Goes to any other court where action the court in new venue uses their
cure want of have jurisdiction could have been brought at the time it state's tort laws. The date it was
jurisdiction was filed. filed in the old venue transfers to
(p. 227) Often been thought of dealing with the new venue, so it is as if the
subject-matter jurisdiction - basic transfer didn't exist.
aspects of the Federal Judicial
Hierarchy. E.g. If lose a case in N

13
District, must appeal to 11th Circuit.
Many cases dealing with 1631 deal with
this issue.

3. Forum Non Conveniens


a. Established under Gilbert case aka "Gilbert Test" common law criteria, no federal statutes.
b. Permits dismissal of case over that a court has jurisdiction and venue over on the basis that
practical factors indicate it should be heard in another court outside the same judicial system.
c. Different from change of venue under 1404 or 1406 b/c venue transfer is okay under those
statutes when venue is within same judicial system.
d. Why? A sometimes is under temptation to resort to a strategy of forcing the trial at a most
inconvenient place for an adversary, at some inconvenience for himself.
e. Two requirements must be satisfied.
1. There must be an adequate alternative forum available for the case.
(a) The applicability of less favorable law will not undermine the status of an alternate
forum as adequate for purposes of the forum non conveniens doctrine [Piper]
2. There must be a showing that interests of convenience to the parties and certain public
interests argue in favor of the alternative forum notwithstanding plaintiffs choice of the
current forum [Gilbert]
f. Litigation convenience factors:
1. Private interest of litigant
2. Ease of access to sources of proof
3. Availability of compulsory process for attendance of unwilling
4. Cost of obtaining attendance of willing (witnesses)
5. Possibility of view of premises
6. Enforceability of a judgment
7. Public interest: distribute litigation equally (no congestion), localized controversies
decided at home, etc.
8. Plaintiff may not by choice of inconvenient forum vex, harass, oppress, the D.
4. Enforcing Forum Selection Clauses

Pleading [Rules 7, 8, 9, 11] & Defenses [Rule 12]


I. Modern Pleading - Complaint
A. Requirements: Each complaint for relief should contain
1. Short statement of the grounds for courts jurisdiction
2. Short statement of the claim showing that pleader is entitled to relief; and
3. A demand for judgment for relief.

14
B. Modern Pleading: Development in recent years [SCOTUS]
1. Plausibility Standard
i. Twombly: To survive dismissal, s complaint must
(a) Contain sufficient factual matter, accepted as true, to state a claim for relief
that nudges claim across line from conceivable plausible on its face (not
just possible)
(b) Plausible on face = when pleads factual content that allows the court to draw
reasonable inference that the is liable for misconduct.
ii. Iqbals 2 Steps
(a) Step 1
(i) Accept as true all non-conclusory factual allegations
(ii) Legal conclusory allegations need not be accepted as true
(b) Step 2 - Plausibility
(i) Do the allegations that must be accepted as true plausibly suggest an
entitlement to relief?
1. Not a "probability requirement"
2. Facts must raise more than a "possibility"
3. Facts must "raise a reasonable expectation that discovery will
reveal [supporting] evidence."
4. Facts that are "merely consistent with" liability are not
sufficient.
5. The inquiry is "content-specific"; the court must "draw on its
judicial experience and common sense."
iii. Example [Ashcroft v. Iqbal]:
(a) P is a Pakistani Muslim who was imprisoned by federal authorities after 9/ 11.
He brings a federal civil-rights action against D1 (the U.S. Attorney General)
and D2 (the FBI Director). He alleges that the two Ds knew and approved of
their subordinates plan to imprison P and hundreds of other Muslim men in
extra-harsh conditions solely on account of the mens race, religion or national
origin, thus violating their constitutional rights. The two Ds move for dismissal
under Rule 12( b)( 6), alleging that even if the subordinates intentionally
violated Ps rights, P has not sufficiently alleged facts showing that the two Ds
acted with a discriminatory purpose (as is required by the substantive rules for
supervisor liability in such cases).
(b) Held, for the Ds: complaint dismissed. Its true that factual allegations in the
complaint must be taken as true. But pure legal conclusions dont qualify
for this taken as true treatment. And those allegations that are truly factual
must plausibly suggest that P is entitled to relief. In determining whether the
complaint meets this plausible suggestion of entitlement to relief standard,
the trial judge is to draw on his judicial experience and common sense.
Here, the complaint does not satisfy the plausibility standard, because even
taking Ps truly-factual allegations as true, the more likely explanation of the
Ds motives is that they were making a bona fide pursuit of national security,

15
not acting for forbidden discriminatory purposes. [Ashcroft v. Iqbal (2009), a
5-4 vote]
iv. Where lacks personal knowledge: where plausibility standard will general make most
difference where alleges facts that he doesnt have personal knowledge and the s
are in sole control of relevant records or testimony. Good examples are cases trying to
hold supervisors liable for violations of civil rights laws [Iqbal] or employment
discrimination laws [Swierkeiwicz].
2. Pleading Special Matters: Limited Exceptions to Rule 8 Standard
i. Rule 9(b) imposes a heightened pleading standard for fraud, mistake, conditions of
mind [malice/intent/knowledge/etc.]
ii. An inference of fraudulent intent may be plausible, yet less cogent than other,
nonculpable explanations for s conduct. Strong = inference of scienter must be more
than merely plausible of reasonable it must be cogent and at least as compelling as
any opposing inference of non-fraudulent intent.
3. Hypo: If you are representing a plaintiff, would you rather be in federal court, or in a state
court that continues to embrace the Conley no set of facts standard? What if you are
representing a defendant?
i. Rep. a - state court that continues to embrace Conley standard b/c less likely motion
of dismissal will be granted b/c of complaint issues.
ii. Rep. a - federal court, b/c the burden on the P to make sure the complaint meets
FRCP and common law standards is greater; if they mess up, have greater chance of Ps
having to amend complaint or having suit dismissed altogether.

II. Responding to Complaint Pre-Answer Motions


A. Pre-Answer Motions Rule 12(b)
1. Alternative to answering a complaint, attacks validity of the complaint.
2. Raise in motion OR in answer [if you want, but not done a lot]
i. Lack of SMJ can be raised at any time.
ii. Lack of PJ
iii. Improper venue
iv. Insufficient process
v. Insufficient service of process
vi. Failure to state a claim upon which relief can be granted; and
vii. Failure to join a party under Rule 19
3. Disfavored defenses:
i. (2) (5)
(a) Lack of PJ
(b) Improper venue
(c) Insufficient process
(d) Insufficient service of process
ii. These are waived from later usage if :
(a) Files a Rule 12 motion that omits the defense; or
(b) Gives an answer that doesnt include the defense and fails to amend it in time.
B. 12(b)(6) Failure to State a Claim upon Which Relief Can be Granted [Why is it so special?]

16
1. Purpose: to test the legal sufficiency of the 's allegations, not their factual truth.
i. Asserts that on the facts as pleaded by [and assumed by the court to be true], no
recovery is possible under any legal theory.
2. Types of 12(b)(6) motions:
i. Pleading doesnt conform to pleading rules [Iqbal; Rule 9(b) motions]
ii. Legally not a cause of action [my bf broke my heart]
(a) Example: If s complaint is barred by the statute of limitations, should
move under 12(b)(6) for failure to state a valid claim. If the court is convinced
that under the facts alleged by , any cause of action would be time-barred, the
court will dismiss.
3. A whos complaint has been dismissed under this rule will virtually always be given at
least one chance to amend claim before it is dismissed.
C. Other Rule 12 Defenses
1. 12(c) Motion for Judgment on the Pleadings
i. cant make motion until after pleadings closed but early enough NOT to delay
trial. But the pleadings arent closed until the answered.
ii. This is why s dont ever use this; use 12(b)(6) instead.
2. 12(e) Motion for a More Definite Statement
i. Before Twombly and Iqbal, 12(e) was common rule for dealing with allegations that
were too vague or conclusory. [used in Garcia v. Hilton Head Hotels, but now could
just use 12(b)(6)]
ii. Function after Twombly and Iqbal? can just dismiss under 12(b)(6) w/o having to let
clarify.
3. 12(f) Motion to Strike
i. Court may strike from pleading insufficient defense or
redundant/immaterial/impertinent/scandalous matter.
ii. Purpose: keeps from having to confirm or deny any of that matter.
iii. On its own or motion by either:
(a) Before responding to the pleading or if response not allowed, or
(b) Within 21 days of being served

III. Answer Rule 8(b), (c); Rule 12


A. state in short and plain terms his defenses to each claim, and admits or denies each count of s
complaint
B. Denials - 8(b)
1. Where not denied: averments [a formal statement by a party in a case of a fact or
circumstance that the party offers to prove or substantiate] in complaint, other than those
relating to damages, are considered admitted if they arent denied. 8(b)(6)
2. Kinds of Denials:
i. General denial: denies every allegation in s complaint @ once. [But must contest
all of s allegations or face sanctions]
ii. Specific denial: denies all of allegations of a certain paragraph or count of complaint.
iii. Qualified denial: denial of particular portion of particular allegation.

17
iv. Denial of knowledge or information (DKI): when says he does not have enough
knowledge or information sufficient to form belief as to truth of s complaint (but
must do this in good faith.)
C. Affirmative Defenses - 8(c)
1. must explicitly plead affirmative defenses in the answer if is going to be allowed to raise
them at trial.
2. Most important are contributory negligence, fraud, res judicata, statute of limitations, and
illegality.
3. General formulation: 8(c) contains more general requirements. must plead affirmatively
any avoidance or affirmative defense any defense that relies on facts particularly within
the defendants knowledge is likely to be categorized as an affirmative defense.
D. Time for Various Pleadings - Rule 12(a)
1. Complaint: filing of complaint occurs before it is served. Service must then occur within 120
days. 4(m)
2. Answer: A must serve answer within 21 days of being served, unless
i. If timely waved service pursuant to 4(d), then gets 60 days after request for waiver sent
by //90 days if intl
ii. If served out of state, by using state long-arm [4(k)(1)(A)], time to answer is
controlled by that state rule.
iii. Rule 12 motion: If makes Rule 12 motion against and loses, has 10 days after
court denies motion to answer.
3. Counterclaim: must be served within 21 days after being served
E. Reply:
1. only required to reply to if ordered, must serve answer within 21 days unless order says
differently.
2. required to reply to affirmative defense b/c automatically considered denying/avoiding
that defense)

IV. Amendment of Pleadings Rule 15(a)


A. As a matter of course: A pleading may be amended once as a matter of course (i.e. without leave of
court) in these circumstances:
1. hasnt already amended it, and if
2. It occurs within 21-day window after pleader served original pleading. 12(a)(1)(A).
3. If responsive pleading is required, amendment of right is also allowed even if it happens
more than 21 days after the original pleading was served, as long as the amendment happens
within 21 days following whichever one is the earlier of (a) service of the responsive
pleading, or (b) service of the other sides motion under Rule 12(b), (e), or (f).
i. This means that when a files a complaint [which requires responsive pleading], the
can amend of right at any time until 21 days after the has served either an answer or
a 12(b), (e), or (f) motion, no matter how much time has elapsed since the served
the original complaint.
B. By leave of court: If situation above categories, i.e. wait too long, may amend only by getting
courts permission. 12(a)(2) says court should freely give leave to amend when justice so requires.

18
1. Actual Prejudice: Leave to amend should be denied only if it would cause actual prejudice to
other party.
2. Amendment after dismissal: If succeeds w/ motion to dismiss original complaint court
will almost always give leave to amend b/c of justice.
C. Time to respond: Any required response to amended pleading should be made within time remaining
to respond to original pleading or within 14 days after service of the amended pleading, whichever is
later.
V. Deterring Frivolous Pleadings Rule 15(a)-(b)

Discovery [Rules 26 37]


I. Scope
A. Parties may discover any material regarding any non-privileged matter that is relevant to any partys
claim or defense.
B. Information can be relevant and discoverable but inadmissible. Discoverable = appears reasonably
calculated to lead to the discovery of admissible evidence"
C. FRCP gives court power to broaden/limit scope.
1. Broaden: for good cause court may permit discovery of any matter relevant to subject
matter of the action.
2. Limit: if discovery sought is unreasonably cumulative or duplicative; obtainable from some
other more convenient, less burdensome, less expensive source; party seeking discovery had
ample opportunity through discovery to obtain info; burden/expense outweighs likely
benefit; importance of proposed discovery in resolving the issues.
i. Can also limit to protect privacy, prevent harassment, prevent undue delay.

II. Devices of Discovery


A. There are 6:
1. Initial Disclosures [Automatic Disclosure]
2. Depositions [written and oral]
3. Interrogatories
4. Document Requests
5. Physical & Mental Exams
6. Requests for Admission of Facts
B. If party refuses to comply w/ discovery request, requesting party must first confer w/ disclosing party
to reach agreement. If no agreement, then disgruntled party may approach the court for an order.
1. If disgruntled party is party seeking information: motion to compel.
2. If disgruntled party is disclosing party: protective order.
C. Conference & Planning for Discovery
1. Timing: Parties must confer as soon as practicable and regardless, at least 21 days before a
scheduling conference is to be held or a scheduling order is due.
2. Content: parties must consider
i. Nature and basis of claims and defenses and possibilities for promptly settling or
resolving the case;
ii. Make or arrange for required disclosures in 26(a)(1);

19
iii. Discuss any issues about preserving discoverable information; and
iv. Develop a proposed discovery plan.
3. Responsibilities: All jointly responsible for arranging conference, for attempting in good
faith to agree on plan, and for submitting to the court within 14 days after the conference a
written report outlining plan.

III. Mandatory Disclosure & the Discovery Plan [Pre-Trial Disclosures]


A. Required Disclosures: a party must [except as exempted by 26(a)(1)(B)] without awaiting a discovery
request, provide to other parties:
1. Name/address/number of each person likely to have discoverable information + subject of
information that disclosing party may use to support its claims/defenses
2. Copy or description by category/location of all docs, ESI, and tangible things that disclosing
party has in its possession, custody, or control and may use to support its claims or defenses
3. Computation of each category of damages claimed by disclosing party. Must make it
available for inspection and copying unless privileged or protected.
4. For inspection and copying, any insurance agreement under which an insurance company
may be liable to satisfy all/part of possible judgment in action or to indemnify/reimburse for
payments made to satisfy the judgment.
B. Time: within 14 days of parties Rule 26(f) conference unless diff. time set.
C. Basis: Party must make initial disclosures based on info then reasonably available to it.
D. Excuses: Party is not excused from disclosing b/c it has not fully investigated, it challenges sufficiency
of another partys disclosures, or because another party hasnt made theirs yet.
E. Supplementing Disclosures and Responses: Once material is disclosed, all parties have a continuing
obligation to supplement their production if
1. Information becomes incomplete due to new information, or
2. If the disclosed information is discovered to be incorrect.
3. Failure to comply = sanctions. Rule 26(g)
IV. Depositions
A. Deposition = compelled, live testimony given under oath; lying = perjury.
B. Procedure
1. Party must give reasonable written notice to every other party, state time and place and
name/address of deponent. This satisfies as obligation for the party to appear.
2. If you want to create an obligation for a non-party to appear and testify, must use a subpoena.
3. Depos only apply to parties!
C. Details
1. Limited to 1 day of 7 hours, but court may authorized more limited time if needed to fairly
examine the deponent or if the deponent, another person, or any other circumstances impedes
or delays the examination. So, some flexibility.
2. Each side is limited to 10 depos.
D. Objections
1. An objection @ time of examination whether to evidence, a partys conduct, the officers
qualifications, the manner of taking the depo, or to any other aspect must be noted on the
record, but the examination still proceeds.
i. The testimony is taken subject to any objection.

20
ii. Must be stated concisely, nonargumentatively
iii. A person may instruct deponent not to answer only when necessary to preserve a
privilege, to enforce a limitation order by court, or present a motion under Rule
30(d)(3).
E. Depositions to Perpetuate Testimony
1. Petition: file verified petition in district court for the district where any expected adverse
party resides.
i. Must ask for order authorizing petitioner to depose the named people to perpetuate
testimony. Must show:
(a) That petitioner expects to be a party to an action in US court but cant
presently bring it or cause it to be brought;
(b) The subject matter of the expected action and the petitioners interest;
(c) The facts that the petitioner wants to est. by the proposed testimony and the
reasons to perpetuate it;
(d) The naems or a description of the persons whom the petitioner expects to be
adverse parties and their addresses as known; and
(e) The name, addressd, and expected substance of the testimony of each
deponent.
2. Order and Examination: If satisfied that perpetuating testimony will prevent failure/delay of
justice, court isues an order depositions taken, etc. See rule.
F. Using Depositions in Court Proceedings
1. Waiver of Objections, to the taking of the deposition.
i. Objection to competence, relevance, or materiality: A failure to make an objection to
one of these before or during the deposition doesnt waive the ability to make the
objection unless the ground for it might have been corrected at that time.
ii. Objection to error or irregularity: waived if
(a) It relates to the manner of taking the dpo, the form of a question or answer, the
oath or affirmation, a partys conduct or other matters that might have been
corrected at that time; and
(b) It is not timely made during the deposition.
iii. Objection to a written question: waived if
(a) Not served in writing on the party submitting the question within the time for
serving responsive questions, or
(b) If the question is a re-cross question, within 7 days after being served with it.
V. Interrogatories to Parties: Rule 33
A. Interrogatory = mailed set of question b/w parties that require answering under oath within specified
time frame.
B. Advantages:
1. Party has duty to respond to interrogatories on basis of her own knowledge and also has to
include what other people know (lawyers, employees, other agents, that reasonably can be
obtained through investigation).
2. Interrogatories allow them to answer the question fully, so you may get information that
helps you more because it is fuller and more complete, and not just one persons perspective
or experiences.

21
3. Advantage over request for documents/ESI interrogatories make some have to find all the
names/content and put them down, whereas if you just wanted documents, thats assuming
what you want exists, and if you were too specific, they dont have to look any further or
obligate someone to find the answer and give to you like you do in interrogatories.
C. Disadvantages: if person doesnt know, then cant answer and you dont get information to help you.
D. Numbers:
1. Respond: 30 days.
2. l
E. Contention Interrogatories:
1. Ask about a partys position (aka contention) re: a specific legal issue. Not just about the
facts, but about how the other party plans to argue it, or what they will claim, etc.
i. Cable v. Computer case FN3 p. 538: each component of damages you claim you have
suffered because of the acts or omissions you allege in the Complaint
2. Can always amend/withdraw, or supplement an answer to interrogatory in court
w/permission
3. Same as all other interrogatories, burden is on the party opposing discovery rather than the
burden be shifted to the proponent of the contention to defend its legitimacy under the FRCP.
4. Explicitly allowed under Rule 22(a)(2) scope. But court can order interrogatory not be
answered until discovery is complete or until a pretrial conference, etc.
VI. Document Requests
A. Can only be gotten from a party! (subpoena for everyone else)
B. Requirements of Request:
1. Must describe with reasonable particularity each item/category of items to be inspected;
2. Must specify a reasonable time, place, and manner for inspection; and
3. May specify the form/forms in which electronic forms and info is to be produced for them.
C. Responses & Objections
1. Time: must respond in writing within 30 days of being served or [if delivered under Rule
26(d)(2)], within 30 days after parties first conference.
2. Responding: For each item or category, response must either state that inspection/etc. will be
permitted as requested or state with specificity the grounds for objecting [with reasons].
i. May state that they will produce copies instead of allowing inspection.
ii. Production then must be completed no later than time for inspection specified or
another reas. time specified in their response.
D. Producing the Docs. or ESI:
1. A party must produce documents as they are kept in usual course of business or must
organize/label them to correspond to request categories;
2. If a request does not specify a form for producing ESI, a party must produce it in a form in
which it is ordinarily kept or in a reasonable usable form.
3. A party need not produce the same ESI in more than one form.
VII. Physical and Mental Exams: Rule 35
A. When the mental or physical condition of a party is in controversy, the court may order the party to
submit to a physical or mental examination by a suitably licensed or certified examiner. *Different
from other discovery requests b/c its not asking for something that already exists, its asking the party
to generate new information by submitting to a new examination.

22
B. Cannot compel a nonparty to submit to a physical/mental examination in any case. [E.g., cannot
compel witnesses to get their eyesight checked, etc.]
C. Requirements:
1. Motion and Good Cause: operates only by court order; must have good cause.
2. Controversy: mental or physical condition of party must be controversy. It is not enough
that the condition would be somehow relevant.
D. Reports from examiner: the actual medical report produced is discoverable.
1. Who may receive: person examined (typically the opposing party) may request a copy from
the party who caused exam.
VIII. Requests for Admission of Truth: Rule 36
A. Scope: A party may serve on any other party a written request to admit, for purposes of the pending
action only, the truth of any matters within the scope of Rule 26(b)(1) related to:
1. Facts, application of law to fact, or opinions about either; and
2. The genuineness of any described documents.
B. Time: Technically, they have 30 days to serve a written answer or objection on the requesting party; a
matter is considered "admitted" if nothing is done after those 30 days are up.
C. Answer: If matter is no admitted, answer must specifically deny it or state in detail why answering
party cannot truthfully admit or deny it.
1. Denial must fairly respond to substance of matter; and when good faith requires that a party
qualify an answer or deny only a part, the answer must specify part admitted and
qualify/deny the rest.
i. Answering party may assert lack of knowledge or info as a reason to failing to admit or
deny only if the party states that it has made reasonably inquiry and that the info it
knows or can readily obtain is insufficient to enable it to admit or deny.
D. Objections: Grounds for objecting must be stated. Cant object solely because the request presents a
genuine issue for trial.
E. Motion re: sufficiency of answer/objection: Requesting party may move to determine sufficiency of an
answer or objection.
1. Unless court finds objection justified, it must order an answer be served.
2. If finds that an answer does not comply, the curt may order a) matter is admitted or b)
amended answer be served.
F. Effect of Admission/Withdrawing or amending:
1. A matter admitted under this rule is conclusively established unless the court, on motion
permits it to be withdrawn or amended.
IX. Subpoena: Rule 45
A. Purpose:
1. Compels a non-party to:
i. Produce documents, ESI, or tangible things;
ii. Attend a deposition
iii. Permit an inspection, etc.
B. If a non-party is to be deposed, then the discovering party can only force the deponent to attend by
issuing a subpoena.
C. This subpoena must require the deposition to be held no more than 100 miles from the place where the
deponent resides, is employed, or regularly transacts business in person.

23
X. Exemptions from Discovery
A. Electronically Stored Information (ESI) in certain situations
1. If particular source reas. accessible b/c undue burden of cost need not provide
disclosure.
2. Requesting party can get order compelling discovery if party refusing to provide discovery
cant show the info is not reas. accessible b/c of undue burden of cost
3. Court can still order discovery even if this undue burden is proven; special conditions may
apply.
B. Privileged Material
1. Only material that is nonprivileged may be discovered.
2. Underlying facts embodied in communications are not protected.
i. Why? If facts could be protected by disclosing to attorney, it would be super easy to
immunize all relevant information from compelled disclosure to other party.
3. Information loses protection if disclosed to third parties not part of the confidential
relationship.
4. 4 General Requirements for Attorney-Client Privilege:
i. The asserted holder of the privilege is (or sought to be) a client;
ii. Communication is made to an attorney (or her subordinate) who is acting as a lawyer
in connection w/ this communication.
iii. The communication is made:
(a) by her client;
(b) in confidence (no strangers present); and
(c) for the primary purpose of obtaining legal advice and not for the purpose of
committing a crime or tort.
iv. The privilege has been claimed and not waived.
C. Work-Product Doctrine
1. Work product = any material prepared in anticipation of litigation or for trial or for another
party or by/for that other partys representative. E.g. notes on witness interviews, recordings
of witness interviews, lists of exhibits, indexes of depos, emails among co-counsel, memos,
etc.
2. Common Law
i. Originally developed in Hickman v. Taylor
ii. Protects from discovery
(a) written statements, private memos, or personal recollections,
(b) prepared or formed by counsel,
(c) about his legal duties.
iii. Exception: But such work product may be discovered if relevant and non-privileged
facts would otherwise remain hidden in an attorney's file; and
iv. Those facts are essential to the preparation of one's case.
3. Work Product Codified: FRCP Rule 26(b)(3).
i. Protects from discovery
(a) documents or tangible things,
(b) prepared in anticipation of litigation,
(c) by or for another party or that party's representative.

24
ii. Exception: does not apply if the party seeking discovery
(a) has substantial need for the materials; and
(b) is unable without due hardship to obtain the substantial equivalent by other
means.
(i) Exception to the exception: even when these requirements are met,
discovery may not be had as to "the mental impressions,
conclusions, opinions, or legal theories of a party's attorney or other
representative concerning the litigation"
iii. Like Hickman, facts are not protected in certain cases, but the party must ask for it
specifically, and not just attorneys files.
(a) I.E., someone can interview the parties themselves to find out what they want
to know, but they cant ask the lawyer what the party told them.
iv. When is it used? E.g. if witness forgets, dies, etc. and getting other parties' lawyer's
notes on the witness statement can be beneficial.
4. Attorney-Client Privilege for organization/corporation
i. The communication must be one that would not have been made but for the
contemplation of legal services;
ii. The content of the communication must relate to the legal services being rendered;
iii. The information-giver must be an employee, agent, or independent contractor with a
significant relationship to the corporation and the corporations involvement in the
transaction that is the subject of legal services;
iv. The communication must be made in confidence; and
v. The privilege may be asserted either by the corporation or by the information-giver.
5. Because of Standard
i. Does not consider whether litigation was a primary or secondary motive behind the
creation of a document. Instead, it considers the totality of the circumstances and
gives protection when it can fairly say the "document was created because of
anticipated litigation, and would not have been created in substantially similar form but
for the prospect of that litigation."
6. Privilege Log: 26(b)(5)(A)
i. Expressly make the claim; and
ii. Describe the nature of the docs, communications, or tangible things not produced or
disclosed.
(a) Enough to enable other parties to assess the claim
7. Clawback: 26(b)(5)(B)
i. Party may notify others that information produced in discovery is privileged or
protected.
ii. If so, the parties who received the information must:
(a) Promptly return, sequester, or destroy the specified information and any copies
it has;
(b) Not use or disclose the information until the claim is resolved; and
(c) Take reasonable steps to retrieve the information if the party disclosed it
before being notified.

25
iii.The receiving party may promptly present the information to the court under seal to
decide wehther it is privileged/protected.
iv. This balances the status quo; receiving party can still dispute whether it is covered by
attorney-client/work-product privilege.
XI. Enforcement of Discovery Rules
A. Sanctions. See 37(b)(2)(A); also
i. Discovery sanctions can both penalize the current parties in the dispute and deter those
who might be in the future tempted to violate the FRCP. (p. 572, Nat'l Hockey League,
Supreme Court, 1976).
ii. Most severe sanction is dismissal of the complaint or ordering entry of judgment
against the offending party;
iii. Also, appointment of a "special master" to oversee the discovery process if it is
continually problematic.
iv. Basis of sanctions = violation of court's order to produce a discovery material.

Summary Judgment: Rule 56


I. Requirements
Movant must show:
1. No genuine dispute as to any material fact; and
2. The movant is entitled to judgment as a matter of law.
Material fact = Fact essential to establishing an element of a claim;
e.g. duty/breach/harm/causation of negligence claim.

A. *Court can also grant SJ sua sponte [on their own, w/o a motion from either party]
B. Court:
1. Must view the facts and draw reasonable inferences in the light most favorable to the party
opposing the summary judgment motion. [Scott v. Harris]
i. Distinction: But only if there is a "genuine" dispute as to those facts: where the record
taken as a whole could not lead a rational trier of fact (jury) to find for the nonmoving
party, there is no 'genuine issue for trial.' [Scott]
2. The evidence of the non-movant is to be believed, and all justifiable inferences are to be
drawn in his favor. The judge uses the substantive evidentiary standard the jury would use at
trial: clear & convincing or preponderance of the evidence. [Anderson]
C. Jury Role:
1. Credibility determinations
2. Weighing of the evidence; and

26
3. Drawing of legitimate inferences from the facts.
4. These are not the judges role. [Anderson]
II. Burden of Production
A. Movant: Initial. Must show no genuine factual dispute. Does not have to offer evidence negating
opposing partys claim, but just have to show other side doesnt have the evidence to prove case or
bother having a trial. It only shifts to nonmovant if the nonmovant would bear the burden of proof at
trial. [Celotex]
B. Nonmovant [if shifted] has several options:
1. Point to specific places in the record* (such as depositions, documents, admissions, etc.)
showing that the fact in question is disputed and the movants claim isnt supported; or
2. Produce affidavits or other evidence.
i. Can show genuine dispute with
(a) Indirect evidence - depends on the assumption that (1) the witness is credible;
and (2) on implicit generalizations;
(b) Direct evidence affidavits, etc.
*Does not have to actually produce new evidence; can refer to evidence in record.
III. Rule 56 v. Rule 12(b)(6)
A. Differences:
1. What sources are considered.
i. 12(b)(6) deals with the initial complaint + legal reasoning alleged by the in the
pleading phase.
ii. Rule 56 motion considers the facts of both sides after discovery [56(c)(1)(A)], and
makes specific references to the substantive facts [that aren't just the allegations in the
complaint] the party thinks make SJ applicable.
2. The court accepts the material fact allegations to be true in a Rule 56 motion after the
movant shows that there is no genuine dispute to any material fact.
IV. In short:
A. Two ways to get summary judgment
1. Adickes route: Must have affirmative evidence that negates an element of the plaintiff's
claim. Must nail down every possible gap through which the plaintiff's claim might be true.
2. Celotex route: Show there is an absence of evidence on the plaintiff's end, and that they
wouldn't be able to recover anyway.

V. Hypos: See Class #23-25 Notes.


VI. Email Questions and Answers:
A. QUESTION: When trying to prove summary judgment by pointing out that the other party lacks
evidence (as in Celotex), does the non-moving party have to bear the burden of proof on that issue at
trial? (Thus, theyve been through discovery and the non-mover bears the burden of proof on that issue,
so if the moving party can still point out the non-moving party does not have enough evidence, the
moving party can win on summary judgment?)
1. ANSWER: Yes. The Celotex "absence of evidence" method only works against a party who
would have the burden of proof on that issue at trial. That is, the non-movant must be the one
who would bear the burden of proof at trial.

27
VII. Summary Judgment Checklist: All About Burdens
A. Should the court enter summary judgment against a party?
1. Movants Party Status is the movant party bearing the burden of proof at trial?
i. Yes. If so, the movant must present the court with sufficient factual evidence [from
existing record or through addl submissions] to support its claim. Proceed to next
question.
ii. No. If not, the movant only has the burden of showing the court that no genuine issue
of material fact exists.
(a) This burden may be discharged through the presentation of affidavits or other
factual evidence or simply by pointing to the existing evidence and arguing
that it fails to support the nonmovants claim.
(b) Once this is done, the nonmovant now has the burden of persuasion; proceed
to next question.
2. Discharging the Burden of Proof has the party bearing the burden of proof at trial pointed
to or presented sufficient factual evidence to support the claim, issue, or affirmative defense
such that SJ should not be entered?
i. Reducible to Admissible Evidence? Has the party carrying the burden of proof
supported its claim with factual information that can be reduced to admissible evidence
at trial? [Note: the info need not be in an admissible form at the SJ stage; rather, it
simply must be information that could be reduced to admissible evidence at a trial]
(a) No. If the party bearing the burden of proof has not supported its claim with
such factual information, the party has failed to meet its burden and SJ should
be granted.
(b) Yes. If so, proceed to the next question.
ii. Persuasive Evidence does the factual information presented by the party bearing the
burden of proof leave some elements of the claim subject to speculation in the face of
more likely lawful explanations of the s challenged conduct? [Matsushita]. Is the
evidence simply the uncorroborated self-interested testimony of the party bearing the
burden of proof?
(a) Yes. If the partys evidence does not establish a plausible, non-speculative
claim particularly in the face of alternate lawful explanations of the s
conduct a court may determine the evidence is insufficient to create a
genuine issue of material fact and enter SJ. Also, the uncorroborated, self-

28
interested testimony alone will not suffice to create a genuine dispute of
material fact.
(b) No. If the partys evidence supports each element of its claim, leaving nothing
to speculation, the SJ should be denied, provided that the claim is established
to the degree required by the applicable standard of proof. Proceed to the next
question.
iii. Standard of Proof does the partys evidence prove its case to the degree required
under the relevant evidentiary standard that would be applicable at trial?
(a) No. If not, SJ should be entered against the party.
(b) Yes. If so, SJ should not be entered against the party

The Civil Action


I. One Form of Action
A. FRCP Rule 2: There is one form of action the civil action.
II. The Jury Trial Right
A. 7th Amendment: In Suits at common law, where the value in controversy shall exceed $20, the right of
trial by jury shall be preserved, and no fact tried by a jury, shall be otherwise re-examined in any Court
of the United States, than according to the rules of the common law.
1. Does not apply to state law hasnt been incorporated against the states
2. Equity provides remedies where the law does not.
B. Test for getting a jury trial:
1. Decide if the statutory action would have been brought in a law/legal court or an equity
court in 18th century England they merged.
i. If law/legal jury.
(a) legal, compensatory damages under codified law.
ii. If equitable no jury.
(a) common law, no legal remedy: the judge has to come up with something.
[Remember: juries only deal with $; we dont trust their judgment to give out
other remedies]
2. Examine remedy sought and determine whether it is legal or equitable in nature.
i. This step is the most important!
C. Jurors
1. Requirements 28 U.S.C. 1865: Any person as long as they are a U.S. citizen; at least 18;
have lived in the district for a year; able to read, write, understand, and speak English; have
no physical or mental infirmity that would render them unable to provide satisfactory service;
convicted of or on trial for a crime that carries a prison sentence of 1+ years and have not had
their rights restored. [Selection controlled by 28 U.S.C. 1861-1865]
2. 2 Step Process for getting jurors
i. List of potential jurors, the venire, is compiled and they are assembled. A number
equal to number that will actually serve are selected at random to sit as a tentative jury.
ii. These tentative juror are questioned by the judge and/or attorneys to determine whether
each of them can decide the case fairly and appropriately. [voir dire.]
(a) If one of them is dismissed, another member of venire takes his place at
random, who in turn gets questioned. Continues until final panel is in place.

29
3. Removing Jurors
i. Challenges for cause permit a prospective juror to be rejected when partiality can be
shown.
ii. Peremptory challenges permit rejection of jurors w/o any statement of reason.
(a) Restrictions differ by state but range from 3-20 per side. In federal system,
each side has 3. Cannot strike someone based on race and sex.

Trial & Post-Trial Motions


I. Burdens of Proof
Burden of Production Burden of Persuasion
"burden of going forward," Jury decides if this is met.
responsible for producing certain threshold If has burden of persuasion:
amt of evidence for claim Already has burden of production; and
Threshold = enough evidence for a Must meet this burden for affirmative
reasonable jury to decide in his favor (but defenses.
what is "reasonable" jury, how does this line Goal: try to convince fact finder (jury or
up with peremptory challenges??) judge) that evidence is more weighty*
can meet burden of production even if all than the other party's. Does this by:
evidence produced is refuted by opposing Casting doubt on
party reliability/credibility of s
Does not ensure victory, but failing to meet it evidence + present own evidence
will guarantee defeat If has Burden of Persuasion, and doesn't
convince jury/judge by standard of proof,
At trial: Judge can grant a motion for judgment jury must rule for .
as a matter of law on burden of production.
*most typical in civil cases: preponderance of
Before trial: Judge can grant Rule 50 MSJ on the evidence [50%] entitled to relief
burdens of production. requested
If total 50/50 toss-up, then should win, b/c
burden of persuasion is to get past 50%.

Judge cannot grant anything on burden of


persuasion; thats jurys job!

This line is up to judge

30
II. Motion for Judgment as a Matter of Law [directed verdict] Rule 50
A. Basics: Allows judge to find that a party has failed to meet its burden of production -- i.e., a reasonable
jury would not have a legally sufficient evidentiary basis to find for the party on that issue. Two
possibilities for deciding party failed to carry burden:
1. fails to present evidence that establishes an essential element; and/or
2. When all the evidence presented is so compelling that the jury would only be permitted to
reach one result as a matter of law.
i. Court guided by substantial evidence test
B. Timing: Can be made at any time after the movants adversary has completed presentation of its case
and before the case is submitted to the jury.
1. Rule 50 provides for motion for JMOL after a jury verdict but only if party making motion
has previously made the motion on the same grounds before the case was submitted to the
jury. This is called
i. Preserving your right:
(a) If you want to motion for JMOL after jury verdict, MUST MOVE TWICE --
(i) First under 50(a)(2) before verdict, and
(ii) Second under 50(b) after verdict, if you dont like it.
(b) Why this structure? Making sure someone deciding as a matter of law doesn't
end up violating 7th amendment this way, the jury still gets to try the facts,
and the movant says ahead of time what the matters of law are in the case.
(i) Court is bound/limited to arguments raised in original pre-verdict
motion for JMOL, so they can't just make up excuses to change jury
verdict.
2. If party fails to make a motion for JMOL before the verdict, the JMOL after the verdict will
be unavailable. [MUST HAVE TWO STEPS!]
3. If the motion is denied, you have 28 days after the jury verdict to ask for a renewed motion.
C. Sua Sponte: Court can enter JMOL s.s. if the party against whom judgment would be entered has been
fully heard on issue.
III. Motion for New Trial Rule 59
A. Basics: Motion may be filed in lieu of or as an alternative to a post-verdict motion for JMOL. But,
different from JMOL in 2 ways:
1. Remedy is a new trial, not a judgment in favor of moving party.
2. Standards for granting a new trial are significantly more flexible than the no reasonable
juror standard used in JMOLs.
B. A new trial can be granted:
1. After a jury trial, for any reason for which a new trial has up to now been granted in an
action at law in fed. court; or

31
2. After a nonjury trial, For any reason for which a rehearing has up to know been granted ina
suit in equity in fed. court.
C. Typical grounds:
1. Errors in jury selection process;
2. Erroneous evidentiary rulings;
3. Erroneous jury instructions;
4. Verdict as being against the weight of the evidence:
i. The judge may consider the credibility of the evidence; unlike JMOL motions, where
judge is asking whether jury could possibly find for the if they believe the evidence,
the judges in new trial motions must ask whether the jury was seriously wrong in
choosing to believe that evidence.
ii. Much more intrusive on jury role as fact finder gives courts more power.
5. Excessiveness or inadequacy of verdict;
6. Misconduct by the judge, attorneys, parties, or witnesses; or
7. Newly discovered evidence.
D. Justification & Criticism:
1. Historical basis justifies allowance under 7th amendment to grant Rule 59 motion for new
trial after jury gives verdict. (If judge decides it's against clear and convincing weight of
evidence).
i. Problematic: can create a cycle until jury finds for the party the judge wants.
ii. Credibility assessments can impact judge's determination to find for a new trial (even
though it's not supposed to, b/c judge supposed to be acting as fact-finder).
E. Judges Role:
1. If substantial evidence in support of , judge may not direct a verdict against him, even
though he may not believe his evidence or may think that weight of evidence is on the other
side b/c it is for the jury to weigh evidence.
i. How they get around this: he can set aside a verdict supported by substantial evidence
where in his opinion it is contrary to the clear weight of the evidence.
2. Important difference b/w trial judge's power on a motion for new trial and on JMOL is that
on a new trial motion he may base his action on his belief or disbelief in some of the
witnesses [effectively weighing evidence], while on a JMOL he may not [consider
witness/evidence credibility].
3. How is this not a review of burden of persuasion? B/c its not a review of 50% line. Gotta be
against the clear weight of the evidence. Less deferential to the jury than burden of
production, but not totally displacing jury w/ respect to burden of persuasion.

Multiple Claims & Parties


I. Permissive Joinder of Claims Rule 18
A. A party asserting a claim, counterclaim, crossclaim, or third-party claim may join, as independent or
alternative claims, as many claims as it has against an opposing party.
B. Limits all obstacles to joinder of claim\s only restriction = [indirect] jurisdictional conditions.
1. Test: Always check for a) venue and b) subject-matter jurisdiction when deciding whether
two claims can be joined!

32
C. A party failing to join such claims is free to raise those claims in a subsequent action.
II. Counterclaims Rule 13(a)-(b)
[drawing]

A. Claim by a against a . [horizontal]


B. Two types: Permissive & Compulsory
1. Compulsory
i. Failure to state: If does not assert a compulsory counterclaim, will lose/waive the
right to bring that claim in any future litigation, even in state court.
(a) E.g., must assert property damage claim in the same claim as personal injury
damage b/c the two damages arise out of the same accident [occurrence].
ii. No limit to type or number of counterclaims a party may bring.
iii. Two Prong / Logical Relationship Test: [M.K. v. Tenet]:
(a) Transactional: they assert any right to relief jointly, severally, or in alternative
w/ respect to or arising out of same transaction, occurrence, or series of
transactions or occurrences; and
(b) Any question of law or fact common to merits of case in action.
iv. Exceptions:
(a) Claims by which just adjudication require the presence of addl parties
that the court cant get personal jurisdiction over; or
(b) The opposing party sued on its claim by attachment [in rem/quasi in rem]
v. Policy:
(a) Courts are interested in:
(i) achieving an efficient use of judicial resources,
(ii) minimizing the burden imposed on litigants, and
(iii) avoiding unnecessary impositions on testifying witnesses.
(b) When it appears that not hearing the two claims together would undermine
these goals by resulting in duplicative litigation, then it is more likely that a
logical relationship between the claims exists.
2. Permissive
i. A pleading may state as a counterclaim against an opposing party any claim that
doesnt rise out of the same transaction or occurrence [i.e., isnt compulsory.]
III. Crossclaims Rule 13(g)
[drawing]

P D1
Counterclaim
Crossclaim
After crossclaim, D1 then has responsibility

33
to bring all counterclaims against D2,
or risk waiving them.

D2
A. Asserted by an already present coparty in the litigation against another co-party. [Vertical, not
horizontal]
B. Test: Proper if
1. Arise out of the same transaction and occurrence that is the subject matter of the original
action;
2. Assert derivative/contingent liability based on claims against the original , if they are the
party bringing the crossclaim.
3. Arise out of a counterclaim; or
4. If claim relates to property that is subject matter in original action.
C. Will generally satisfy supplemental jurisdiction b/c related to same subject matter as original action.
D. Remember: Crossclaims are permissive, but once a crossclaim is asserted, the party against whom the
crossclaim is asserted is now in an adversarial relationship with the asserter. They now have to deal
with compulsory counterclaims must assert any/all they have or risk waiving them in the future.

IV. Third-party Claim Rule 14(a)(1)-(5), (b)


A. can drag in another party and serve them because new party may be liable to the current for all or
part of the claim against the [derivative claim, contribution, indemnity, etc.]
1. Timing: if complaint is filed >14 days after serving its original answer, must obtain the
courts leave.
2. Duties of Third-party:
i. Must assert any defense against 3rd party s claim under Rule 12;
ii. Must assert any compulsory counterclaim against 3rd party ;
iii. May assert any permissive counterclaim; and
iv. May assert any crossclaim against another 3rd party .
3. Test: Same Two-Prong transaction-subject matter as counterclaims.
4. Plaintiffs: When a claim is asserted against a , they can bring in a third party if this rule
would allow the to do so.
B. Any party may move to strike a third-party claim, sever, or try it separately.
C. The third-party may also bring in another nonparty to be a third-party 2 who is or may be liable to
the third-party for all or part of any claim against it. Can go on and on and on.
D. Remember: Consider whether the jurisdiction comparative or JSL impacts whether or not Rule 14
third-party claim is applicable.
1. Comparative Fault - only pays its share anyway, no need to bring in a third party to pay for
indemnity.
2. JSL - Can get around it and essentially do the same thing using Rule 13 Counter/Crossclaims
+ Rule 20 Permissive Joinder of Parties.
E. Consider the following scenario: A is injured in a bus accident. A sues B, the manufacturer of the
bus, alleging that there was a defect in the buss design.

1. Basic Finger-Pointing

34
B may seek to avoid liability to A by asserting that the accident was actually caused by
some other actor, e.g., faulty maintenance by C, the bus operator. Generally, B is able
to do this even if C is not a party to the case. B can present evidence about Cs
culpability and can argue to the judge or jury that C is at fault without C being
formally brought into the case as a party. In this situation, B is not arguing that C must
pay anything to A or that C must pay anything to B. If C is not a party, the court cannot
order C to pay anything to anyone. B is simply seeking to avoid paying anything to A,
on the theory that C (not B) is responsible.

What happens if both B and C bear some responsibility for As injuries? There are a
number of ways that the substantive law handles this situation. Two common methods
are joint and several liability and comparative fault.

2. Joint & Several Liability


Under this concept, a defendant who is only partially liable for the plaintiffs injuries
is liable for all of the plaintiffs damages. So B would have to pay 100% of As
damages even if B is only partially responsible for the accident (and C is responsible
for the remainder) *what remainder??*

Joint and several liability systems typically allow a defendant who is held liable to seek
what is called contribution from others who share responsibility. If B pays 100% of
As damages because of joint and several liability, B would have a contribution claim
against C; if B establishes in its contribution claim that C is 50% responsible, C will
have to compensate B for 50% of what B paid to A.

3. Comparative Fault
Under this concept, the judge or jury in A v. B would decide what Bs share of liability
is, and B would be obligated to pay A only Bs share of the damages. If B is found
50% responsible, B would pay 50% of As total damages. In the bus scenario, B might
use basic finger-pointing (see above) to establish that C is partially responsible for the
accident. In a comparative fault system, this would reduce Bs liability to A even if C
is not a party to the case.

A comparative fault scheme typically does not allow B to seek contribution from C,
because B has paid only its share of the damages. For A to collect the remainder of her
damages, she would have to sue C directly.

V. Joinder of Parties Rules 19, 20, 21


A. Permissive Joinder Rule 20:
1. Two Types

35
i. Joinder of s: Two s may voluntarily join together in action if satisfy two prongs:
(a) Same transaction or occurrence; and
(b) Common question of law or fact to all s in action
ii. Joinder of s: If 1+ have a claim against multiple defendants, the s can be joined
(a) If joint, several, or alternative relief asserted against them by the (s) arising
out of the same transaction, occurrence, or series of transactions, occurrences
(b) Any question of law or fact common to all s will arise in the action
iii. Neither nor has to be interested in obtaining/defending against ALL the relief
demanded.
iv. Joinder of s is at option of .
2. REMEMBER: Personal & subject matter jurisdiction + venue must still be met.
B. Compulsory Joinder Rule 19: for situations where the decision of the court might require binding of
a party who was not in the court.
1. First step: Court decides if absent party is a required party under Rule 19(a)(1).
i. A required party is considered required in 3 situations:
(a) In that persons absence, the court cannot give complete relief among the
existing parties;
(i) E.g.) Assume A sues B to recover property he sold to B and C on the
ground of misrepresentation. Because C is a co-owner of the
property, if C is not joined, A will not be able to recover the
property without initiating a second action against C. A thus cannot
obtain complete relief solely in its action against B. C = required
party in this example.
(b) That person claims an interest relating to the subject of the action and is
situated that disposing of the action w/o them may impair/impede the persons
ability to protect the interest; or
(i) E.g.) Assume A sues B seeking payment of money from a limited
fund. If C is similarly entitled to payment from that fund, but a
judgment in A v. B would deplete the fund, Cs future ability to
collect on her claim would be impaired. C could thus be considered
a necessary party.
(c) That person claims an interest relating to the subject of the action and is
situated that disposing of the action w/o them may leave an existing party
subject to a substantial risk of incurring double, multiple, or otherwise
inconsistent obligations because of the interest.
(i) E.g.) Assume A sues B for possession of land. B leases the land
from C. If A prevails in the action against B, B will lose possession
of the land but will still be obligated to pay rent to C. Thus, C should
be treated as a necessary party to avoid the possibility that B would
be subjected to two inconsistent obligations in this situation.
ii. Non-required party example is a joint tortfeasor.
(a) E.g.) In a situation where A alleges negligence against B (the manufacturer of
an allegedly defective medical device), C (the doctor), and D (the hospital),

36
that were involved in the operation installing the device in A would be
considered joint tortfeasors that need not be joined under Rule 19.
(b) SCOTUS has held that joint tortfeasors are simply permissive parties; it is not
necessary for all joint tortfeasors to be named as defendants in a single lawsuit
[Temple v. Synthes]
2. Second Step: Court then decides if joinder of that required party is feasible.
i. Joinder of a required party is not feasible in 3 situations:
(a) No PJ;
(b) No subject-matter jurisdiction [typically occurs b/c joining party destroys
diversity];
(c) Lack venue, provided the prospective party objects to venue.
3. If party is required and joinder is not feasible, court can determine whether, in equity and
good conscience, the action should proceed or be dismissed.
i. Factors include:
(a) Extent to which a judgment rendered in that persons absence might prejudice
that person or existing parties;
(b) The extent to which any prejudice could be lessened or avoided by:
(i) Protective provisions in judgment;
(ii) Shaping the relief; or
(iii) Other measures;
(c) Whether a judgment rendered in persons absence would be adequate; and
(d) Whether the could have an adequate remedy if the action were dismissed for
nonjoinder.
C. Misjoinder and Nonjoinder of Parties Rule 20
1. Misjoinder of parties is not a ground for dismissing an action. On motion or sua sponte, the
court may at any time, on just terms, add or drop a party or sever any claim against a party.
VI. Intervention
A. Intervention of right: On timely motion the court must permit anyone to intervene who:
1. Is given an unconditional right to intervene by fed. statute; or
2. Claims an interest relating to the property or transaction that is the subject of the action, and
so is situated that disposing of the action may as a practical matter impair or impede the
movants ability to protect its interest, unless existing parties adequately represent that
interest.
i. Test for Adequacy of Representation:
(a) A putative intervenor does not have an interest not adequately represented by
a party to a lawsuit simply because it has a motive to litigate that is different
from the motive of an existing party. P. 448; Nat. Resources Defense v.
NYState Dept. of Environ. Conservation
(b) So long as the party has demonstrated sufficient motivation to litigate
vigorously and to present all colorable contentions, a district judge does not
exceed the bounds of discretion by concuding that interest of intervenor is
adequately represented.
(c) Motivation Interest; can have different motivations yet share the same
interest in the suit.

37
B. Permissive Intervention:
1. In general: On timely motion, the court may permit anyone to intervene who:
i. Is given a condl right to intervene by fed. state; or
ii. Has a claim or defense that share w/ the main action a common question of law or
fact.
2. Third parties can intervene after a judgment on the merits, provided the conditions in FRCP
24(a) are satisfied.

Advanced Subject Matter Jurisdiction


I. Supplemental Jurisdiction 28 U.S.C. 1367
A. A state-law claim lacking an independent basis for federal jurisdiction may be heard in federal court on
the basis of supplemental jurisdiction when it is part of the same case or controversy as a claim in the
case that does have its own basis for federal jurisdiction (the freestanding claim). The Supreme Court
has defined this as meaning that the claims arise from a common nucleus of operative fact.
B. Things to remember:
1. When can state and federal claims be combined in fed. court? When they derive from a
common nucleus of operate fact! So, I only care about 1367 if I don't have a federal law
hook for subject matter jurisdiction.
2. IF THE ORIGINAL CLAIM ISNT BASED ON DIVERSITY ALONE, DONT EVEN
LOOK AT 1367(b)! DONT LOOK AT IT. YOU HAVE SUPP. JXN.
3. A defendant who brings a third-party claim, counterclaim or a crossclaim is not a "plaintiff"
under 1367(b).
C. 2 Steps:
1. Step 1: Original Jurisdiction [1367(a)]
i. Is there a basis for federal subject matter jxn [e.g. 1331 or 1332] over some claim in
the case?
(a) The answer must be yes for supplemental jxn to exist.
2. Step 2: Same case or controversy [1367(a)]
i. Is the claim that lacks an independent basis for federal subject matter jxn related
enough to the claim from Step 1 that they form part of the same case or controversy
under Article III? Refer to Gibbs Test.
(a) Answer must be yes for supplemental jxn to exist.
3. Excluded Claims: [1367(b)]
i. 1367(b) forbids supplemental jxn if
(a) The original jurisdiction hook [see Step 1] is 1332; and
(b) The claim for which supplemental jxn is invoked is either:
(i) By a against persons made parties under Rule 14, 19, 20, or 24; or
(ii) By a person proposed to be joined as a plaintiff under Rule 19; or
(iii) By a person seeking to intervene as a plaintiff under Rule 24.
(c) When exercising supplemental jurisdiction over such claims would be
INCONSISTENT WITH JURISDICTIONAL REQUIREMENT OSF
SECTION 1332.

38
D. Court Discretion: Courts may decline to give sup. jxn over a claim that would normally meet
1367(a)s criteria [2 Steps] if
1. The claim raises a novel or complex issue of State law;
2. The claim substantial predominates over the claim or claims over which district court has
original jurisdiction;
3. The district court has dismissed all claims over which it has original jxn; or
4. In exceptional circumstances, other compelling reasons.
E. Example where AIC in 1332 not satisfied, but sup jxn exists: Exxon Mobil v. Allapattah (N5, p.
239-240): Court considered whether sup. Jxn may be exercised over s joined w/ Rule 23 or Rule 20
when complete diversity of citizenship is satisfied and the claims of the joined parties are related to
claims over which diversity jurisdiction exists, but do not meet the amt-in-controversy requirement.
1. Court held that where (1) other elements of jurisdiction are present and (2) at least one in
the action satisfies AIC requirement, 1367 does authorize sup. Jxn over the claims of other
s in the same Article III case or controversy even if those claims are for less than 75K.
F. Email Questions & Answers:
1. QUESTION: Gibbs Case and Takeaway?
i. ANSWER: What used to be called "pendent jurisdiction" is now governed by the
supplemental jurisdiction statute (28 USC 1367), but pendent jurisdiction case law like
Gibbs may still be relevant for two reasons.
(a) FIRST: Most importantly, Gibbs' common nucleus of operative fact language
informs whether the claim satisfies 1367(a)'s requirement that the claims "form
part of the same case or controversy under Article III of the United States
Constitution." SECOND: Assuming that supplemental jurisdiction is OK under
1367(a) & (b), the block quote you cite from Gibbs sheds light on some of the
considerations that might inform whether a court should nonetheless decline to
exercise supplemental jurisdiction over a particular state-law claim. Many of
these are also explicitly codified in 1367(c) itself, but you are correct that this
is ultimately discretionary so courts might also invoke the considerations
highlighted in Gibbs (and different courts take different approaches on this).
With respect to trying a state law claim even though the federal law claim is
dismissed before trial, 1367(c) makes this a discretionary decision for the
district court.
ii. Complete diversity is not required if the basis for subject matter jurisdiction is 1331.
But if the basis for subject matter jurisdiction would be 1332, a lack of complete
diversity means 1332 is not satisfied, which destroys the basis for original jurisdiction
(and hence leaves nothing to which supplemental jurisdiction might attach).
2. QUESTION: If A is from AL and is suing B from CA in federal court and the hook for being
there is 1332, can B bring in C who is also from California or will that destroy diversity?
Can we try and get supplemental jurisdiction? Would we have to have a federal question, can
we do a Rule 14/18 strategy or a 13/20 one?
i. ANSWER: If B brings a third-party claim against C, that would not destroy complete
diversity. And if Bs claim against C is a federal question, then there's an independent
basis for subject matter jurisdiction and you don't need to worry about 1367. If B's
claim against C does not create a federal question, then you would need to use 1367

39
because there's no diversity as between B and C. Assuming the claim by B against C
forms part of the same case or controversy for purposes of Article III (as required by
1367(a)), 1367(b) would not be an obstacle because B's claim is by a defendant, not a
plaintiff.
3. QUESTION: Does the part that says "when exercising supplemental jurisdiction over such
claims would be inconsistent with the jurisdictional requirements of section 1332" apply to
the whole part of the last prong or just those that seek to intervene in Rule 24? So, if a
plaintiff brings someone in through Rule 20, is he allowed to do this as long as the person
does not ruin diversity or are all joiners under Rule 20 (assuming plaintiffs original hook is
diversity) barred?
i. ANSWER: That clause applies to the entire section, but as a practical matter it doesn't
affect the ultimate analysis because you only need supplemental jurisdiction (1367) in
cases where 1332 would not be satisfied as to that claim.
ii. That doesn't necessarily fix the problem with respect to Rule 20, though. If the parties
being brought in under Rule 20 are additional plaintiffs, then 1367(b) does not forbid
supplemental jurisdiction (See the Exxon case discussed in Note 5 on pp.239-240). But
if the plaintiff is using Rule 20 to sue multiple defendants, and 1332 is the basis for
original jurisdiction, then the earlier language in 1367(b) would forbid supplemental
jurisdiction: that would be a claim "by [a] plaintiff[] against persons made parties
under Rule 14, 19, 20, or 24 of the Federal Rules of Civil Procedure."
II. Removal - 1441(a)-(c) 's opportunity to invoke federal subject matter jurisdiction.
A. In General: A in a state court may have the entire case sent aka removed, to the federal district court
geographically in the location of the state court if the case could have been brought in federal court
originally.
1. This means the standards governing federal subject matter jxn are relevant to figuring out
whether removal is allowed. It also means that removal can only be based on the claims,
removal is not possible based on the s counterclaims or defenses.
i. Why? Mottley! Because a federal law defense to a state law cause of action does not
create 1331 jurisdiction and therefore that action is not one that fed. courts have
jurisdiction over. Also, defense wasnt brought when original jurisdiction was decided,
aka when federal subject matter jxn was decided, which is based on Mottley!
2. When claims qualify for diversity jurisdiction, or some other type of fed. jxn, the entire case
may be removed to federal court.
B. Restrictions
1. Only s may remove a case!!!!!!!! Why???
i. Ensures s as well as s have opportunity to decide which cases fed. courts hear.
ii. Protects out-of-state from threat of local bias [has an opportunity to go to save haven
of fed. court]
iii. But doesnt always align w/ these idealistic goals.
2. Case may not be removed on basis of diversity jxn if any is a citizen of the forum state. If
multi-defendant suit, even one in-state defendant blocks removal. Why? B/c that defeats the
whole purpose of having fed. jxn eliminating bias, etc. No point of protection if is from
the same state as the state court. So, if you want to keep from removing: join a party from
the state as a (permissive)!!!

40
3. All s must agree to removal.
4. Only possible if s comply w/ time limits in statute: 30 days after receipt of service of
complaint.
C. Procedurally
1. removes case simply by filing a notice of removal with the fed. court for the
district/division geographically embracing the state court.
2. If a party believes removal is erroneous violation of 1441 or lack of SMJ can then move
in the fed. court under 1447(c) to remand the issue back to state court. The motion must be
made within 30 days of the filing of notice of removal.
D. Perks of being in federal court for s:
1. Potential state-court bias; it allows for s to also help decide what cases federal courts hear
versus just the plaintiffs having a say (Venue, Transfer, etc.).
2. Personality of federal judges also, whether the judge has personal bias or is known to rule a
certain way in general or for specific issues [think philosophy, patience for certain types of
cases, etc.)
3. Pleading Standards: Twombly/Iqbal very friendly interpretations of FRCP; many state
courts
4. Forum non conveniens: Piper v. Reno [fed. Courts have framework for granting this for cases
to proceed outside US, a lot of state courts have not embraced forum non conveniens]
5. Summary judgment: state courts are often more friendly towards .
6. Discovery: Might be more strict in state courts [proportionality]; might be able to get more
info from in fed. Court.
E. Hypos:
1. Hypo1: Your client is an Alabama citizen injured in Alabama while driving a car
manufactured by Guster Motors, Inc. (a Delaware corporation with its principal place of
business in Michigan). You believe that the crash was caused by a defect in the cars braking
system. If you want to sue Guster in a state court and prevent removal, what could you do?
What if you want to sue in an Alabama state court?
i. If no other party, but want to sue Guster in state court and prevent removal: I would
bring the case in a Michigan or Delaware state court - even if it seems inconvenient, it
would ensure that he can't remove it. That's when 1441(b)(2) comes into play have
complete diversity, seeking more than 74K, but (b)(2) doesn't let Guster remove b/c he
is a from the state in which the claim was filed.
(a) ***A cannot remove a case from one of its own state courts to federal court.
Why? B/c then you wouldn't have concern of out-of-state litigants being biased
by state court. Guster motors does not need protection from MI/Delaware state
courts. This is why we don't have a rule like this for federal question
jurisdiction -- which is about their expertise in questions of federal law. That
stuff doesn't hinge on whether you have home state defendant.
ii. If another possible party: Could get a from Alabama [maybe driver of the other car],
which would destroy complete diversity.
iii. However, if you decide to sue under Federal Act, open up HUGE door for to
remove could use 1331 [which doesn't depend on diversity] to remove. Want to
be sure to use state law claim not a federal statute.

41
iv. In Alabama state court, just would allege damages for less than $75,000.
2. Hypo2: ABC Corp., a Delaware corporation with its principal place of business in Maryland
wants to sue two members of its board of directors, Mike and Jennifer, for fraud. Mike is a
citizen of Virginia and Jennifer is a citizen of New York. ABC claims $75,000 in damages
against Mike and $75,000 in damages against Jennifer, because each of them separately
defrauded the company out of $75,000. Can ABC bring its case in federal court?
i. There is no claim based on federal law here, so jurisdiction will depend on whether
there is diversity under 1332(a). The first requirement is that the parties be completely
divers. ABC is considered domiciled in the state it is incorporated and its principal
place of business, which is Delaware and Maryland respectively. Both the s are from
two separate states Virginia and New York. Thus, none of the parties on either side
share a common state with the other. Next, the amount in controversy rule in 1332(a)
must be satisfied. The AIC must exceed 75K. The is alleging 75K for each of the s,
but the claims are completely individual, so no JSL, and thus, no diversity. No federal
court jurisdiction.
3. Hypo3: Consider 28 U.S.C. 1441(c) and Note 8 on p.243. If a state court action contains
both a non-diverse state law claim and an unrelated federal claim, can the defendant remove?
What will happen if the defendant does remove?
i. Yes - because there is an unrelated federal claim. If the does remove, the district
court shall sever the non-diverse state law claim and remand it to the State court from
which it was originally removed from.
ii. Note 8 Question: Is removal blocked if s state court action joins a nondiverse state
law claim with an unrelated federal claim?
(a) No, removal is not blocked - it just goes forward with only the federal claim
and the non-diverse state law claim is remanded to the state court in which it
was brought in.
(b) Constitutional significance of requiring district court to sever and remand? It
make sure the federal courts do not overstep their bounds and impede on the
sovereignty of the state courts.
Preclusion
I. Basics: Two Types: Claim & Issue Preclusion
A. Always at least 2 cases
1. Case 1 = case that causes the preclusion
2. Case 2 = the case that will be affected by preclusion.
B. Always ask: To what extent does Case 1 have preclusive effect on Case 2?
C. See chart below for tests.

42
Claim When Case 2 is brought, judgment on Case 1s entire suit will be considered
Preclusion conclusive, as to matters that actually were litigated or should have been litigated
[res judicata]: in the first suit. Three elements.
1. Only judgments that are final, valid and on the merits have preclusive
effect.
2. Judgment in Case 1 is b/w the same parties involved in Case 2 (or those in
privity with those parties).
o This is the most important difference between issue preclusion.
3. The claim in the second suit must involve matters properly considered
included in the first action.
o What was the claim litigated in Case 1?
o That same claim cannot be relitigated in Case 2.
*No issue requirement, even as to issue that weren't actually decided in first case.
Critical Under claim preclusion, claim may be "merged" or "barred" by party's
difference: failure to raise the claim in a prior action.
Issue preclusion, however, applies only to matters actually argued and
decided in an earlier lawsuit.
Issue A right, question, or fact distinctly put in issue and directly determined by court of
Preclusion correct jurisdiction cannot be disputed in subsequent suit between the same parties
[collateral or their privies, even if the second suit is for a different cause of action. The
estoppel]: right/question/fact determined must be taken as conclusively established, so long
as judgment in first suit is unmodified.

1. The judgment in case 1 is final, valid, and on merits..


unless the issue for which preclusion is sought is a procedural issue
(which means that the judgment does not have to be "on the merits")
2. The same issue is involved in Case 1 and Case 2
3. That issue was actually litigated in Case 1.
4. Determination of that issue was necessary to the Case 1 judgment.
5. [Others?]
Issue was important
The judgment in Case 1 was b/w same parties involved in Case 2
["mutuality" requirement]

43
II.

44

You might also like